Q26. If \( a \in \mathbb{R} \), then the equations \( x^2 + x + a = 0 \) and \( x^2 + a x + 1 = 0 \) have a common real root for:
Solution: We are given two equations: \[ x^2 + x + a = 0 \quad \text{and} \quad x^2 + ax + 1 = 0, \] with a common root \( \alpha \). Substituting \( \alpha \) into both equations: 1. From \( x^2 + x + a = 0 \): \( \alpha^2 + \alpha + a = 0 \), 2. From \( x^2 + ax + 1 = 0 \): \( \alpha^2 + a\alpha + 1 = 0 \). Subtracting these equations: \[ (a - 1)(\alpha - 1) = 0. \] This gives two cases: - If \( a = 1 \), the first equation has no real roots since \( \Delta < 0 \). - If \( \alpha = 1 \), substituting into the first equation gives \( a = -2 \). For \( a = -2 \), \( x = 1 \) is a common root. Thus, \( a = -2 \) is the only solution in the interval \( [-3, -1] \). Therefore, the correct answer is **Option C**.
Interactive Math Quiz - Question 298
Q28. Let the equations of two circles \( C_1 \) and \( C_2 \) be given by \[ x^2 + y^2 - 4x - 4y + 6 = 0 \] and \[ x^2 + y^2 - 10x - 10y + k = 0, \] where \( k \) is a constant. Suppose that \( C_1 \) and \( C_2 \) have exactly two common tangents. Then possible values of \( k \) are:
Solution: Centre and radius of a circle \( x^2 + y^2 + 2gx + 2fy + c = 0 \) are \( (-g, -f) \) and \( \sqrt{g^2 + f^2 - c} \), respectively.
For \( C_1 \): Centre = \( (2, 2) \), Radius = \( \sqrt{2} \).
For \( C_2 \): Centre = \( (5, 5) \), Radius = \( \sqrt{50 - k} \).
Distance between the centres = \( \sqrt{(5 - 2)^2 + (5 - 2)^2} = \sqrt{18} \).
According to the properties of circles:
If two circles have exactly two common tangents, then \[ |\text{Difference of radii}| < \text{Distance between centres} < |\text{Sum of radii}| \] Substituting: \[ \sqrt{50 - k} - \sqrt{2} < \sqrt{18} < \sqrt{50 - k} + \sqrt{2}. \]
Analyzing options: - **Option A**: For \( k = 17 \), \( |\sqrt{7} - \sqrt{2}| < \sqrt{18} < |\sqrt{7} + \sqrt{2}| \). Does not satisfy. - **Option B**: For \( k = 25 \), \( |\sqrt{5} - \sqrt{2}| < \sqrt{18} < |\sqrt{5} + \sqrt{2}| \). Satisfies. - **Option C**: For \( k = 49 \), \( |\sqrt{1} - \sqrt{2}| < \sqrt{18} < |\sqrt{1} + \sqrt{2}| \). Does not satisfy.
Therefore, the correct answer is **Option B**.
Interactive Math Quiz - Question 299
Q29. Consider the function \[ f(x) = \begin{cases} 2x - 1 & \text{if } x < -1 \\ x^2 + 1 & \text{if } -1 \leq x \leq 1 \\ x + 1 & \text{if } x > 1 \end{cases} \] Then
Solution: The given function is: \[ f(x) = \begin{cases} 2x - 1 & \text{if } x < -1 \\ x^2 + 1 & \text{if } -1 \leq x \leq 1 \\ x + 1 & \text{if } x > 1 \end{cases} \]
**At \( x = -1 \):**
\[ \lim_{x \to -1^-} (2x - 1) = 2(-1) - 1 = -3 \] \[ \lim_{x \to -1^+} (x^2 + 1) = (-1)^2 + 1 = 2 \] Since the left-hand limit (\(-3\)) and the right-hand limit (\(2\)) are not equal, \( f(x) \) is **discontinuous at \( x = -1 \)**.
**At \( x = 1 \):**
\[ \lim_{x \to 1^-} (x^2 + 1) = (1)^2 + 1 = 2 \] \[ \lim_{x \to 1^+} (x + 1) = 1 + 1 = 2 \] Since the left-hand limit and the right-hand limit are equal, \( f(x) \) is **continuous at \( x = 1 \)**.
Therefore, \( f(x) \) is continuous everywhere except at \( x = -1 \).
The correct answer is **Option C**.
Interactive Math Quiz - Question 300
Q30. The sum of the first 50 terms of the series \( 3+7+13+21+31+43+\ldots \) is:
Solution: The given series is \( 3, 7, 13, 21, 31, 43, \ldots \), and the differences between consecutive terms are: \[ 4, 6, 8, 10, \ldots \] which form an arithmetic progression with a common difference of 2.
Let the \( n \)-th term of the series be \( T_n = an^2 + bn + c \). Using the first three terms: - For \( n=1, T_1=3 \): \( a(1)^2 + b(1) + c = 3 \), so \( a + b + c = 3 \). - For \( n=2, T_2=7 \): \( a(2)^2 + b(2) + c = 7 \), so \( 4a + 2b + c = 7 \). - For \( n=3, T_3=13 \): \( a(3)^2 + b(3) + c = 13 \), so \( 9a + 3b + c = 13 \).
Solving these equations gives: \[ a = 1, \, b = 1, \, c = 1 \] Hence, the general term is: \[ T_n = n^2 + n + 1 \] The sum of the first \( n \) terms is: \[ S_n = \sum_{n=1}^N T_n = \sum_{n=1}^N (n^2 + n + 1) \] Using the sum of squares and the sum of integers, we get: \[ S_n = \frac{N^3 + 3N^2 + 5N}{3} \] Substituting \( N = 50 \): \[ S_{50} = \frac{50^3 + 3(50)^2 + 5(50)}{3} = 50 \times 885 \] Therefore, the final answer is **D: \( 50 \times 885 \)**.
Interactive Math Quiz - Question 301
Q31. If \[ A_{n}=\frac{\sum_{k=1}^{n} k(k+1)(k+2)}{n \cdot \sum_{k=1}^{n} k(k+1)} \] then \[ \lim_{n \to \infty} A_n \] is:
Solution: The expression for \( A_n \) is: \[ A_n = \frac{\sum_{k=1}^n k(k+1)(k+2)}{n \cdot \sum_{k=1}^n k(k+1)} \]
Numerator: Expanding \( \sum_{k=1}^n k(k+1)(k+2) \): \[ \sum_{k=1}^n k(k+1)(k+2) = \sum_{k=1}^n \left(k^3 + 3k^2 + 2k\right) \] Using summation formulas: - \( \sum_{k=1}^n k^3 = \left(\frac{n(n+1)}{2}\right)^2 \sim \frac{n^4}{4} \) (dominant term), - \( \sum_{k=1}^n k^2 = \frac{n(n+1)(2n+1)}{6} \sim \frac{n^3}{3} \) (dominant term), - \( \sum_{k=1}^n k = \frac{n(n+1)}{2} \sim \frac{n^2}{2} \). Thus: \[ \sum_{k=1}^n k(k+1)(k+2) \sim \frac{n^4}{4}. \]
Denominator: Expanding \( \sum_{k=1}^n k(k+1) \): \[ \sum_{k=1}^n k(k+1) = \sum_{k=1}^n \left(k^2 + k\right) \] Using summation formulas: - \( \sum_{k=1}^n k^2 \sim \frac{n^3}{3} \), - \( \sum_{k=1}^n k \sim \frac{n^2}{2} \). Thus: \[ \sum_{k=1}^n k(k+1) \sim \frac{n^3}{3}. \] Multiplying by \( n \): \[ n \cdot \sum_{k=1}^n k(k+1) \sim \frac{n^4}{3}. \]
Final Simplification: For large \( n \): \[ A_n = \frac{\frac{n^4}{4}}{\frac{n^4}{3}} = \frac{3}{4}. \] Therefore: \[ \lim_{n \to \infty} A_n = \frac{3}{4}. \] The correct answer is **A**.
Interactive Math Quiz - Question 302
Q32. The function \( f : \mathbb{R} \to \mathbb{R} \), defined by \( f(x) = x^3 - 3x^2 + 6x - 5 \), is:
Solution: Step 1: Differentiate \( f(x) \) to Analyze Monotonicity To determine if the function is one-to-one, we examine its derivative: \[ f'(x) = 3x^2 - 6x + 6 = 3\left((x-1)^2 + 1\right) \] Since \( (x-1)^2 + 1 > 0 \) for all \( x \in \mathbb{R} \), we conclude: \[ f'(x) > 0 \quad \forall x \in \mathbb{R} \] This means that \( f(x) \) is strictly increasing on \( \mathbb{R} \), so the function is one-to-one (injective).
Step 2: Analyze the Range of \( f(x) \) Since \( f(x) \) is a cubic polynomial and is strictly increasing, its range is: \[ f(x) \in (-\infty, \infty) \] Thus, \( f(x) \) is onto (surjective) because every real number is covered in the range.
Step 3: Conclusion - The function \( f(x) \) is one-to-one because it is strictly increasing.
- The function \( f(x) \) is onto because its range covers all real numbers.
Therefore, the correct answer is
one-to-one and onto.
Interactive Math Quiz - Question 303
Q33. The number of distinct words that can be formed using all the letters except vowels of the word
'PROBABILITY' is:
Solution: The word 'PROBABILITY' contains the following letters: \[ \text{P, R, O, B, A, B, I, L, I, T, Y} \] Removing the vowels (O, A, I, I), we are left with the consonants: \[ \text{P, R, B, B, L, T, Y} \] The total number of permutations of these letters, accounting for the repetition of B (which appears twice), is given by: \[ \frac{7!}{2!} = \frac{5040}{2} = 2520 \] Hence, the number of distinct words that can be formed is \( \mathbf{2520} \).
Final Answer: A.
Interactive Math Quiz - Question 304
Q34. The area enclosed between the curves \( y = 2x^2 \) and \( y = 6 \) is:
Solution: The area enclosed between the two curves \( y_1 = f(x) \) and \( y_2 = g(x) \) is calculated as: \[ A = \left| \int_{x_1}^{x_2} \left( f(x) - g(x) \right) dx \right| \] Here, \( f(x) = 6 \), \( g(x) = 2x^2 \), and the limits of integration are found by solving \( 6 = 2x^2 \): \[ x^2 = 3 \quad \Rightarrow \quad x = \pm \sqrt{3} \] Substituting these values, the area becomes: \[ A = \left| \int_{-\sqrt{3}}^{\sqrt{3}} \left(6 - 2x^2 \right) dx \right| \] Compute the integral: \[ A = \left| \left[ 6x - \frac{2x^3}{3} \right]_{-\sqrt{3}}^{\sqrt{3}} \right| \] Evaluate the definite integral: \[ A = \left| \left[ 6\sqrt{3} - \frac{2(\sqrt{3})^3}{3} \right] - \left[ -6\sqrt{3} + \frac{2(-\sqrt{3})^3}{3} \right] \right| \] Simplify: \[ A = |[-4\sqrt{3}] - [4\sqrt{3}]| = 8\sqrt{3} \]
Final Answer: \( \mathbf{8\sqrt{3}} \).
Correct Option: D.
Interactive Math Quiz - Question 305
Q35. The value of \( \lim_{x \to 0} \frac{\sin(x^2)}{x \sin x} \) is:
Solution: We know that \( \lim_{x \to 0} \frac{\sin x}{x} = 1 \). Using this property: \[ \lim_{x \to 0} \frac{\sin(x^2)}{x \sin x} = \lim_{x \to 0} \left( \frac{\sin(x^2)}{x^2} \cdot \frac{x}{\sin x} \right) \] - For the first term: \( \lim_{x \to 0} \frac{\sin(x^2)}{x^2} = 1 \) because \( \sin(x^2) \approx x^2 \) as \( x \to 0 \). - For the second term: \( \lim_{x \to 0} \frac{x}{\sin x} = 1 \) because \( \sin x \approx x \) as \( x \to 0 \). Combining these: \[ \lim_{x \to 0} \frac{\sin(x^2)}{x \sin x} = 1 \cdot 1 = 1 \]
Final Answer: \( \mathbf{1} \).
Correct Option: B.
Interactive Math Quiz - Question 306
Q36. The value of \( \frac{30_{C_{1}}}{2} + \frac{30_{C_{3}}}{4} + \frac{30_{C_{5}}}{6} + \ldots + \frac{30_{C_{29}}}{30} \) is:
Q37. In the quadrilateral ABCD below, \( \angle DAB = 90^\circ \) and \( AB = 24 \, \text{cm} \), \( BC = 24 \, \text{cm} \), \( CD = 50 \, \text{cm} \), and \( AD = 18 \, \text{cm} \) (The diagram is not drawn to scale). Find the area of the quadrilateral.
Solution: Join \( BD \). The area of quadrilateral ABCD is the sum of areas of triangles ABD and BCD.
In \( \triangle ABD \), applying Pythagoras' Theorem: \[ AB^2 + AD^2 = BD^2 \quad \Rightarrow \quad 24^2 + 18^2 = BD^2 \quad \Rightarrow \quad BD = 30 \] Also, in \( \triangle BCD \), applying Pythagoras' Theorem: \[ BD^2 + BC^2 = CD^2 \quad \Rightarrow \quad 30^2 + 40^2 = 50^2 \] This confirms that \( \triangle BCD \) is a right-angled triangle.
Now, calculate the areas: - Area of \( \triangle ABD \) is: \[ \text{Area of } \triangle ABD = \frac{1}{2} \times AB \times AD = \frac{1}{2} \times 24 \times 18 = 216 \] - Area of \( \triangle BCD \) is: \[ \text{Area of } \triangle BCD = \frac{1}{2} \times BD \times BC = \frac{1}{2} \times 30 \times 40 = 600 \] Therefore, the total area of quadrilateral ABCD is: \[ \text{Area of ABCD} = 216 + 600 = 816 \]
Final Answer: 816 \( \text{cm}^2 \).
Correct Option: C.
Interactive Math Quiz - Question 308
Q38. Let \( x = \frac{\pi}{40} \). Then the value of \( \cot x \cot 2x \cot 3x \ldots \cot 19x \) is:
Solution: The expression is: \[ \cot\left(\frac{\pi}{40}\right) \cot\left(\frac{2\pi}{40}\right) \ldots \cot\left(\frac{19\pi}{40}\right) \] Now, we know that: \[ \cot A = \tan\left(\frac{\pi}{2} - A\right) \] Thus, \[ \cot \frac{\pi}{20} = \tan\left(\frac{\pi}{2} - \frac{\pi}{40}\right) = \tan \frac{19\pi}{40} \] Also, \[ (\cot A)(\tan A) = 1 \] Hence, all the terms other than \( \cot \frac{10\pi}{40} \) pair up and become 1. Additionally, \[ \cot \frac{10\pi}{40} = \cot \frac{\pi}{4} = 1 \] Therefore, the value of the expression is: \[ 1 \]
Final Answer: 1.
Correct Option: A.
Interactive Math Quiz - Question 309
Q39. Consider the function \( f(x) = |2 - |x - 1|| \) for all \( x \in \mathbb{R} \). Then the value of \( f^{\prime}(-2) + f^{\prime}(0) + f^{\prime}(2) + f^{\prime}(4) \) is:
Solution: Consider the function \( f(x) = |2 - |x - 1|| \). We analyze it piecewise as follows: \[ |x - 1| = \begin{cases} x - 1 & \text{if } x \geq 1 \\ 1 - x & \text{if } x < 1 \end{cases} \] Hence, \( f(x) \) will be defined piecewise based on the value of \( x \): \[ f(x) = \begin{cases} |x + 1| & \text{if } x < 1 \\ |3 - x| & \text{if } x \geq 1 \end{cases} \] The derivative \( f'(x) \) will depend on the piecewise definition, and the values for \( f'(x) \) at specific points are: \[ f^{\prime}(-2) = -1, \quad f^{\prime}(0) = 1, \quad f^{\prime}(2) = 1, \quad f^{\prime}(4) = -1 \] Therefore, \[ f^{\prime}(-2) + f^{\prime}(0) + f^{\prime}(2) + f^{\prime}(4) = -1 + 1 + 1 - 1 = 0 \] Hence, the correct answer is: \[ \boxed{0} \]
Final Answer: B.
Correct Option: B.
Interactive Math Quiz - Question 310
Q40. Let \[ P = \begin{bmatrix} a & b & 0 \\ -1 & 2 & 1 \\ 2 & -3 & -2 \end{bmatrix} \] with \( \det(P) = -2 \). Then the minor \( M_{22} \) of \( P \) is:
Solution: Given matrix \( P \): \[ P = \begin{bmatrix} a & b & 0 \\ -1 & 2 & 1 \\ 2 & -3 & -2 \end{bmatrix} \] The determinant \( \det(P) = -2 \). We calculate the minor \( M_{22} \) of \( P \) by eliminating the second row and second column: \[ M_{22} = \det \left[ \begin{array}{cc} a & 0 \\ 2 & -2 \end{array} \right] \] The determinant of this \( 2 \times 2 \) matrix is: \[ \det \left[ \begin{array}{cc} 2 & 0 \\ 2 & -2 \end{array} \right] = 2(-2) - 0(2) = -4 \] Therefore, the minor \( M_{22} \) is \( -4 \).
Final Answer: A.
Correct Option: A.
Interactive Math Quiz - Question 311
Q41. If \( \alpha \) and \( \beta \) are the two roots of the equation \( x^2 + x + 1 = 0 \), then the value of \( \alpha^{2017} + \beta^{2017} \) is:
Solution: Consider the equation \( x^3 - 1 = 0 \). This can be factored as: \[ x^3 - 1 = (x - 1)(x^2 + x + 1) = 0 \] The roots are \( 1, \alpha, \beta \), where \( \alpha \) and \( \beta \) are the roots of \( x^2 + x + 1 = 0 \). Since \( \alpha^3 = 1 \) and \( \beta^3 = 1 \), it follows that: \[ \alpha^{2017} = (\alpha^3)^{672} \cdot \alpha = \alpha, \quad \beta^{2017} = (\beta^3)^{672} \cdot \beta = \beta \] Thus: \[ \alpha^{2017} + \beta^{2017} = \alpha + \beta \] From the properties of the roots of the equation \( x^3 - 1 = 0 \), we know that: \[ 1 + \alpha + \beta = 0 \Rightarrow \alpha + \beta = -1 \] Hence, the value of \( \alpha^{2017} + \beta^{2017} = -1 \).
Final Answer: C.
Correct Option: C.
Interactive Math Quiz - Question 312
Q42. The number of different solutions \((x, y, z)\) of the equation \(x + y + z = 10\), where \(x\), \(y\), and \(z\) are positive integers, is:
Solution: Assume \(x = a + 1\), \(y = b + 1\), and \(z = c + 1\), where \(a\), \(b\), and \(c\) are non-negative integers. Substituting in the equation: \[ (a+1) + (b+1) + (c+1) = 10 \quad \Rightarrow \quad a + b + c = 7 \] The number of non-negative integer solutions for \(a\), \(b\), and \(c\) is given by the formula for the number of solutions to the equation: \[ \binom{7 + 3 - 1}{3 - 1} = \binom{9}{2} = \frac{9 \times 8}{2} = 36 \] Therefore, the number of different solutions is \(36\). The correct answer is A.
Final Answer: A.
Correct Option: A.
Interactive Math Quiz - Question 313
Q43. In the xy-plane, the equation \(x^2 - y^2 = 2y + 1\) represents a:
Solution: We start with the equation: \[ x^2 - y^2 = 2y + 1 \] Rearranging terms, we get: \[ x^2 = y^2 + 2y + 1 \] Which can be rewritten as: \[ x^2 - (y + 1)^2 = 0 \] Factoring the difference of squares, we get: \[ (x - y - 1)(x + y + 1) = 0 \] This represents two straight lines: \[ x - y - 1 = 0 \quad \text{and} \quad x + y + 1 = 0 \] Therefore, the equation represents a pair of straight lines. The correct answer is D.
Final Answer: D
Interactive Math Quiz - Question 314
Q44. There are 100 students in a class. In an examination, 50 of them failed in Mathematics, 45 failed in Physics, and 40 failed in Biology. 32 failed in exactly two of the three subjects. Only one student passed in all the subjects. The number of students failing in all three subjects is:
Solution: Let the number of students who failed in exactly 1 subject be \( x \),
the number of students who failed in exactly 2 subjects be \( y \),
and the number of students who failed in all 3 subjects be \( z \).
We have the following system of equations:
x + 2y + 3z = 50 + 45 + 40 = 135
x + y + z = 100 - 1 = 99
Subtracting the second equation from the first:
y + 2z = 36
It is given that \( y = 32 \), so:
2z = 36 - 32 = 4
z = 2
Thus, the number of students failing in all three subjects is 2.
Final Answer: C
Interactive Math Quiz - Question 316
Q46. An equilateral triangle, having each side as \( a \), has its corners cut away so as to form a regular hexagon. The area of the hexagon is:
Solution: To make a regular hexagon, three equilateral triangles of side \( \frac{a}{3} \) will be cut from the corners to form a regular hexagon of side \( a \).
Hence, this hexagon can be further divided into 6 equilateral triangles of side \( \frac{a}{3} \).
The area of the hexagon is:
\[ 6 \cdot \frac{\sqrt{3}}{4} \cdot \left( \frac{a}{3} \right)^2 = \frac{\sqrt{3} a^2}{6} \]
Hence, the answer is \( A \).
Interactive Math Quiz - Question 317
Q47. Let \( f(x) = a_0 + a_1 |x| + a_2 |x|^2 + a_3 |x|^3 \), where \( a_0, a_1, a_2 \) and \( a_3 \) are constants. Which of the following statements is correct?
Solution: For the function to be differentiable at \( x = 0 \), the derivative must satisfy: \[ f'(0) = \lim_{h \to 0} \frac{f(h) - f(0)}{h} \] Now using the value of \( f(x) \), we get: \[ f'(0) = \lim_{h \to 0} \frac{a_0 + a_1|h| + a_2|h|^2 + a_3|h|^3 - a_0}{h} \] Simplifying further: \[ f'(0) = \lim_{h \to 0} \frac{a_1|h| + a_2|h|^2 + a_3|h|^3}{h} \] For \( h < 0 \), \( f'(0) = -a_1 + a_2 h - a_3 h^2 \).
For \( h > 0 \), \( f'(0) = a_1 + a_2 h + a_3 h^2 \).
For the two expressions to match as \( h \to 0 \), it must be that \( a_1 = 0 \). Hence, the correct answer is \( C \).
Interactive Math Quiz - Question 319
Q49. Let \( S = \{1, 2, \dots, 100\} \). The number of nonempty subsets \( T \) of \( S \) such that the product of numbers in \( T \) is even is
Solution: The number of ways to select a non-empty set from \( S = \{1, 2, \dots, 100\} \) is: \[ \binom{100}{1} + \binom{100}{2} + \dots + \binom{100}{100} = 2^{100} - 1 \] Similarly, the number of ways to select a non-empty set from the odd numbers in \( S \) (i.e., \( P = \{1, 3, 5, \dots, 99\} \)) is: \[ 2^{50} - 1 \] Hence, the required number of sets is: \[ 2^{100} - 1 - (2^{50} - 1) = 2^{50}(2^{50} - 1) \] Therefore, the correct answer is \( A \).